7
$\begingroup$

Let $\mathcal{A}:X\to Y$ be continuous linear operator, $X$ and $Y$ are Banach spaces. Let $\text{Im} \mathcal{A}=Y$.

Is $\ker\mathcal{A}$ a complemented subspace of $X$?

1 Answers 1

8

No, not in general. For example, if $X$ is not isomorphic to a Hilbert space then $X$ contains a noncomplemented subspace $Z$. The quotient map $X \longrightarrow X/Z$ is surjective, continuous and linear, but the kernel (which is $Z$ of course) is not complemented.

  • 0
    A minor issue: you probably need $Z$ closed here, as $Y$ should be Banach.2012-02-28
  • 0
    @Philip Brooker As far as I know this result is due to Lindenstrauss and Tzafriri. Given a normed space X which is not isomorphic to Hilbert space, how to construct non-complementable subspace?2012-02-28
  • 1
    Probably the easiest example of a non-complemented subspace of a Banach space is $c_0 \subset \ell^\infty$. [This short note](http://dx.doi.org/10.2307/2315346) by Whitley in the Monthly gives a very short proof. So you can take $Z = c_0$ and $X = \ell^{\infty}$ to give a specific example.2012-02-28
  • 0
    No, I just wanted to get more or less constructive algorithm for arbitrary space $X$ which is not isomorphis to Hilbert space. Anyway thanks for example2012-02-28
  • 0
    @Leandro: you are right that $Z$ is closed. However, in practice (i.e. in journal articles) Banach space theorists usually omit the word 'closed' and simply write 'subspace' whenever they mean 'closed subspace'; well, that is my experience anyway.2012-02-28
  • 0
    @Norbert, I do not know an algorithm as such off the top of my head, but then I can't remember the details of the Lindenstrauss-Tzafriri result, so I don't know if it is likely that such a thing exists. I'll have a look at it when I get a chance (quite busy today).2012-02-28
  • 0
    Thanks, for your attention. You will have some progress there I will make a distinct question, which you can answer.2012-02-28
  • 0
    @Norbert: My previous comment wasn't directed at you :) Anyway, this crucial lemma in the proof of the result (from the old LNM 338 edition of Lindenstrauss-Tzafriri) amounts to such a recipe or "algorithm", I think: [page 1](http://i.stack.imgur.com/o56H9.png), [page 2](http://i.stack.imgur.com/yyxtN.png).2012-03-01
  • 0
    @t.b. Thanks for the reference!2012-03-01
  • 1
    I would like to point you to another question which is related to this one: http://math.stackexchange.com/questions/2119896/can-every-closed-subspace-be-realized-as-kernel-of-a-bounded-linear-operator-fro If the restriction of this question to the case that $X=Y$ would result in a positive answer, this would result in a negative answer to the question I have linked above. @Norbert2017-02-01